This is a MBT question, since the question stem specifies: The statements above logically commit the politician to which one of the following conclusions?

Our stimulus begins with the conditional indicator unless. If we translate this by negating one of the ideas and making it the sufficient condition, we should get something equivalent to: If our nation does not redistribute wealth, then we cannot alleviate economic injustice AND eventually there will be intolerable economic inequities. This dovetails nicely into the next sentence’s conditional, which tells us that intolerable economic inequities is enough for those who suffer from injustice to resort to violence. Having now given us this set of conditionals about the dire consequences of not redistributing wealth, the politician concludes with a rule; it is our nation’s responsibility to do whatever is necessary to alleviate conditions that lead to violent attempts at social reform.

It should be clear how this rule relates to what we are already told. Based on the preceding two sentences we know that not redistributing wealth means there will be intolerable economic inequities which will in turn necessarily lead to violent attempts at social reform. The lack of redistribution is therefore a condition which necessarily gives rise to violent attempts at social reform, triggering the politicians rule, and consequently it must be true that it is our nation’s responsibility to redistribute wealth.

Answer Choice (A) We haven’t been told anything about whether violent attempts at reform can be justified, we only know that they inevitably arise under certain conditions.

Correct Answer Choice (B) This is exactly what we should have noticed in our pre-phrasing. Whenever the stimulus of a MBT question contains a conditional rule, we should be thinking about whether the sufficient condition is triggered anywhere else in the stimulus.

Answer Choice (C) The stimulus tells us nothing about political expediency and whether it must be chosen over abstract moral principles.

Answer Choice (D) This is consistent with our politician’s statements, but only because they deal with specifically a case that will lead to intolerable social conditions, and mention nothing about what to do if they don’t. We can’t conclude from what the politician says that economic injustice isn’t a problem in other cases.

Answer Choice (E) This is a classic case of confusing necessity for sufficiency. Just because redistribution is required for economic justice does not mean it is enough for economic justice.


Comment on this

We’ve got a MBT question here, as the question stem asks: If the statements in the argument are all true, which one of the following must also be true on the basis of them?

This stimulus is all about constitutions and what can make them liberal. It begins by denying the view that whether a constitution is written or unwritten inherently makes them more or less liberal. The big takeaway here should be that just knowing a constitution is written isn’t enough to know whether or not it is liberal. The next sentence further emphasizes this point with the conditional indicator until; a written constitution by itself is just a piece of paper, it only becomes meaningful when its content is (i) interpreted and (ii) applied. Put otherwise, if a written constitution hasn’t been interpreted and applied, then it isn’t a real constitution yet. These two sentences arguing against the importance of whether a constitution is written are followed up by a helpful definition of what a constitution is. A constitution isn’t the written or unwritten text, it is a sum of procedures which together legitimize and control the state’s use of power. Our final sentence begins with the conclusion indicator therefore, and from all this talk about what a constitution is and whether its being written matters, concludes that a written constitution is only liberal if it is (i) interpreted and (ii) applied in a liberal way.

This is a four star question, and it is easy to see why; unlike a lot of MBT true questions the underlying logic of the stimulus isn’t made super clear. The key to getting this question correct is recognizing the essential claims the stimulus makes about what makes something a constitution and what makes something a specifically liberal constitution. The key triggers are whether a constitution has been interpreted and applied, and whether this was in a liberal way. Let’s look at the answer choices and eliminate the ones that could be false:

Answer Choice (A) Since we’ve been told a written constitution is just a paper with words until it is put into action, it must be false that we could judge whether a written constitution is actually liberal just by reading it.

Correct Answer Choice (B) B is correct for the same reason that A is incorrect. It says the opposite of A, and tells us that a written constitution can’t be judged to be liberal by its written text alone, which is exactly what must be true given the definition of a constitution and the requirements to be liberal that the stimulus gives us. If a written constitution is just words on paper prior to being interpreted and applied, then we can’t know whether it is a liberal constitution without it being interpreted and applied.

Answer Choice (C) This answer makes a really broad claim that we have zero support for in the stimulus. All we’ve been told is that written constitutions are not inherently more liberal than unwritten ones because a constitution becomes real only through interpretation and application. We know nothing about whether or not there are advantages between the two kinds.

Answer Choice (D) We were only told that there is nothing inherently liberal about written constitutions, we cannot infer from that that there is something inherently liberal about unwritten constitutions.

Answer Choice (E) This answer choice is the most selected incorrect answer for this question. We are given the conditional statement that if a written constitution is liberal, then it is required that it be interpreted and applied in a liberal way. What this answer does which makes it incorrect is confuse the sufficient and necessary conditions of this statement, so that merely being interpreted in a liberal way is enough for a constitution to be liberal. If you chose this answer, it might be helpful to review conditional logic and specifically why we cannot conclude that jsut because the necessary condition is true that the sufficient condition is as well.


Comment on this

This is a must be true, indicated by: If all of the statements in the above are true, which one of the following statements must also be true?

In the stimulus we are given three sentences that can be converted into conditionals. Reasonable → adapt to the world. Unreasonable → try to adapt world to self. Progress → unreasonable people. The correct answer is going to be something which is guaranteed by these conditionals, their contrapositives, or their combinations. Let’s find it:

Answer Choice (A) Even if the information in the stimulus could guarantee that unreasonable and reasonable people are incompatible in a particular way, such as maybe in how they adapt to the world, it still could not guarantee that they are just flat out incompatible in general.

Correct Answer Choice (B) This answer depends on properly recognizing the contrapositive of “All progress depends on unreasonable people”. Depends means requires, and therefore we can think of this as saying unreasonable people are a requirement of progress. If there are only reasonable people, there are no unreasonable people, and since those are a requirement of progress, then there can be no progress.

Answer Choice (C) This answer makes the classic mistake of confusing sufficiency for necessity. Just because there is no progress without unreasonable people, does not mean there are no unreasonable people without progress. If you selected it, you should review this from the curriculum.

Answer Choice (D) Although we cannot conclude that because all progress requires unreasonable people, all unreasonable people create progress, neither can we conclude that some unreasonable people cannot create progress. All we get from the third sentence is the conditional “if there is progress, then there must be unreasonable people”, and the contrapositive “if there are no unreasonable people, there can be no progress.” What proportion of unreasonable people actually bring about progress is unknown to us.

Answer Choice (E) Just because unreasonable people persist in trying to make the world fit them, doesn’t mean they are more persistent overall than reasonable people. The stimulus tells us nothing about the persistence of reasonable people.


Comment on this

This is a must be true question, as it asks: If the statements above are true, which one of the following must also be true on the basis of them?

This stimulus is full of conditionals with comparisons. The first sentence and the second half of the second sentence both begin with the conditional indicator “when”, which gives us three conditionals in addition to the “but if” beginning the second sentence. All three conditionals involve comparisons indicated by “than” or “as”. If we notice that the “fall more slowly” of the necessary condition of the first conditional is equivalent to the “fall less rapidly” of the second conditional, and that the “unable to lower prices” of the second conditional is equivalent to the “cannot lower prices” of the third, we should recognize that the three conditionals form a chain: slower adoption → slower falling costs → cannot lower prices → squeezed out. The contrapositive of this chain is: not squeezed out → can lower prices → not slower falling costs → not slower adoption. Since this is a must be true question with a chain of conditionals, we should be looking for answers which say something about a condition earlier in this chain or the contrapositive chain that guarantee a condition further down the chain. Let’s see if any answers take this form:

Answer Choice (A) We are told nothing about raising prices, only stuff about being unable to lower them.

Answer Choice (B) This answer is a classic case of confusing sufficiency for necessity. Just because foreign competitors (FCs) adopting technologies faster is sufficient to squeeze a country out of the global market, doesn’t mean that it is required for a country to be squeezed out of the market.

Answer Choice (C) This answer makes the same mistake as B, just with different parts of our chain of conditionals.

Answer Choice (D) The problem with this answer is that it gives us the negation of the first condition of our conditional chain, because “the same rate” is equivalent to “not slower”, but we can only use the negation in the contrapositive where it is the final necessary condition. Even if A→B is true, we can’t infer anything from B alone. In this case we certainly cannot infer that neither group will be squeezed out of the market. It is entirely consistent with what we are told in the stimulus that there are a million ways a manufacturer can be squeezed out of the market even when it has the same tech adoption rate as its FCs.

Correct Answer Choice (E) Our contrapositive chain comes in handy here. If we look at it, we’ll notice that if a manufacturer can lower prices as rapidly as their foreign competitor, then they must not have slower falling costs, which means that they must not be adopting tech at slower rate. If it is true that a manufacture can lower prices as rapidly as its FCs, then it is required that it is not adopting tech at a slower rate.


1 comment

This is a strengthen question, and we know that because of the stem: Which one of the following, if true, most strengthens the argument?

Our stimulus tells us that asbestos (which they also include a nice little description of) poses health risks only if it’s disturbed and released into the environment. The author says that since removing it from buildings would disturb it, the government should not require the removal of all asbestos. This argument, as it stands, makes sense. You wouldn’t want to remove asbestos is disturbing it would cause harm. Our instinct is to be alarmed that anyone would support the idea of leaving asbestos in our walls and just living it. This argument assumes that the alternative (removing and putting it somewhere else) is worse.

Answer Choice (A) Plugging this back into the argument does not help the argument. We’re pointing out that asbestos is not as dangerous as all these other things, but that doesn’t strengthen the idea that the government should not require its removal.

Answer Choice (B) This is a conditional statement: if workers do not wear protective gear, asbestos can pose a health threat. What about if they do wear protective gear? Would there still be a threat? We don’t know! In other for this to interact with the stimulus, we would need to know that the workers are not wearing protective gear at the very least.

Answer Choice (C) These relative statements are meaningless: how much more? And is the less dangerous kind of asbestos dangerous in general? At what level? It could be that one is very dangerous, and the other is extremely dangerous. We’ve already said that asbestos, when disturbed, is dangerous - this relative statement does not do anything to strengthen the argument.

Answer Choice (D) This could potentially weaken the stimulus. They’re saying that since the asbestos will eventually get disturbed, what’s the point of preventing its removal now? This answer choice is essentially saying this restriction is unnecessary.

Answer Choice (E) This answer choice draws out a potential alternative proposal for removing asbestos and saying that it’s potentially dangerous: if we require that people remove all asbestos and then put it in a landfill (we can assume a landfill would be where opponents would want to dumb the asbestos), it’s still not safe.


1 comment

We’ve got a MSS question, which we can tell from the question stem: The statements above most strongly supports the conclusion that comparative advertisements

The stimulus begins with an common opinion of marketing experts about what a company should do in a nonexpanding market. Apparently the best strategy is to go after a bigger share of the market. This makes some sense; if the market as a whole (superset) isn’t expanding, then if an individual company (subset) wants to grow it should try to make up a larger portion of the market. According to these marketing experts, the best way to accomplish this is to run ads comparing your products to rivals and emphasizing their weaknesses. What this first sentence does is gives us a strategy for a specific situation, that of a company in a market that isn’t growing.

The first thing that you should notice in the next sentence is that we are talking about a stagnant market, the food oil market. Something is stagnant if it isn’t changing, so this market is exactly the type of nonexpanding one which the marketing experts strategy applies to. For two years the soybean oil and palm-oil did what marketing experts believe works; for two years they ran advertisements comparing products and emphasizing the bad health effects of each other. Contrary to the marketing experts’ belief, this strategy had little effect on the market shares of either side, and instead many consumers stopped buying edible oil (i.e food oil) at all. Instead of either company (subset) growing, their ads discouraged consumers from buying food oils, strongly suggesting that the food oil market (superset) actually shrunk. Let’s see what strongly supported inference we can make from this information:

Answer Choice (A) Nothing in the stimulus mentions relative superiority between products. We are told about a strategy marketing experts believe in, and then given an example where the strategy didn’t succeed.

Answer Choice (B) The marketing experts advice specifically applied to nonexpanding markets, and our example was of one of these markets. This answer wants you to assume that because a strategy was recommended for specific type of market, it shouldn’t be used outside that market. The problem is that we don’t know anything about how effective comparative advertisements are in those other markets. Maybe even though in this particular nonexpanding market the strategy backfired, it would actually work great in some expanding markets.

Answer Choice (C) This answer choice might be appealing because the one example we are given involved a battle between two companies that led to negative consequences for both sides. The problem is that this answer tries to draw a universal conclusion from a particular instance. Maybe in the car market comparative ads used in retaliation are really effective. We generally want to avoid answers that try to derive an absolute rule from just one example.

Correct Answer Choice (D) D is correct because it avoids the error of C. Where C made a claim that in all cases retaliatory comparative ads are a bad idea, C only says that they might risk a market contraction. It’s important that we recognized the overall decline of the food oil market mentioned in the last sentence of the stimulus, because it tells us that comparative advertising in our example did lead to a contraction of the food oil market. While deriving an absolute rule from a particular case is poor reasoning, a particular case is always good evidence for the mere possibility of a thing. If it rained one day that’s bad evidence that it will rain every day, but good evidence that it can rain.

Answer Choice (E) The example we were given didn’t mention any verification on consumers part. Maybe they did verify the claims about health effects made by both sides and that’s why they stopped buying either product.


Comment on this

This is a Resolve, Reconcile, and Explain Question. We know this because of the question stem: “...most helps to resolve the apparent discrepancy?”

RRE questions will require an explanation of a conflicting set of facts. Our correct answer choice, when plugged back into the stimulus, will resolve the discrepancy by explaining how two sides of the apparent conflicting issues actually make sense together. The correct answer will use both sides, though not necessarily explicitly, to explain the conflict. Often, the test will entice you to make naive assumptions about the conflict - don’t fall for it! Your approach should fall under the “this seems wrong because of xyz, but I can think of a few reasons it could work.”

Our first sentence presents us with what happens when you consume excess calories: you gain weight. This makes sense. Our next sentence says that alcoholic beverages have tons of calories. This (sadly) tracks.

Before we read on, with these two ideas in mind, what can we infer? That excess intake of alcoholic drinks could cause someone to gain weight. With this question, it’s possible to anticipate where the argument could go; they’re probably going to say something about how, contrary to these facts, people who drink a lot of alcohol don’t gain weight.

What does our next sentence say? Exactly this: people who drink 2-3 drinks, exceeding their intake, do not generally gain weight. The grammar here is a little complicated. If helpful, focus on the subject and then the predicate and then expand your scope from there. The subject is “people.” What kind of people? And so on.

Back to the argument. Can we think of reasons why people who drink 2-3 beverages a day do not gain weight? Perhaps these people exercise a lot. Perhaps these people have higher metabolisms. These are just two reasons - they are many, many more.

Okay - let’s go to the answer choices.

Answer Choice (A) This may seem attractive, but it’s absolutely wrong. We’re directly rejecting the facts we’re given. The people who drink these excess calories do exceed their caloric intake - it’s in the last sentence. This is out.

Correct Answer Choice (B) It’s giving an overlooked possibility for how these people who drink 2-3 drinks are able to not gain weight. The excess calories are dissipated through heat, which means they’re not converted and stored as fat.

Answer Choice (C) This answer choice focuses on people who do not drink but who eat high-calorie foods and still do not gain weight. How does this help resolve the fact that people who drink excess calories don’t gain weight?

Answer Choice (D) This is a restatement of the last sentence! It doesn’t add or explain anything.

Answer Choice (E) This is completely irrelevant to our issue: people who don’t eat excess calories don’t lose weight. Don’t lose weight? This doesn’t even interact with either of our sides.


Comment on this

This page shows a recording of a live class. We're working hard to create our standard, concise explanation videos for the questions in this PrepTest. Thank you for your patience!

Here we have a five star question with a peculiar question stem. We should at least know that this is a strengthening question because it asks: Which one of the following ethical criteria, if valid, would serve to support the journalist’s conclusion. The question adds another requirement of a correct answer, stating it will support the conclusion “while placing the least constraint on the flow of reported information.” An extra qualification can throw you off, but thankfully this isn’t something you see on LR questions anymore. The LSAT writer wants this requirement to lurk in the back of your mind and distract you, but when an extra requirement is added, you should focus on approaching the question as normal and only bring in the second requirement if you have multiple answers which meet the first.

The stimulus tells us about a journalist in a civil war who found evidence that the government was responsible for refugees starving. Unfortunately, government censors removed any mention of the government from the journalist’s report. The journalist concluded (alarm bells should be going off for you here) that it was ethically permissible to file the censored report, because (more alarm bells) it would be preceded by a notice that it had gone through government censors. This is a rare case where the stimulus is more straightforward than the question stem.

The key sentence is the last one, it gives us the conclusion which a correct answer must support, and the journalist’s reasoning for it. She believes that although the report had been altered by the government, it is still ethically permissible to publish the censored version if readers are explicitly told it had been cleared by government censors. The correct answer will give us an ethical principle which support’s the journalist’s reasoning, while incorrect answers will fail to do this or even outright contradict the journalist’s conclusion. Let’s move to the answers now:

Answer Choice (A) The first thing we should do when judging this answer is ignore “it is ethical in general to report known facts”, because it is a broad statement which adds very little to justify the journalist’s decision. What we are interested in is the rest of the answer, which only tells us what is unethical. The “if” should jump at you, and if we convert this answer to a conditional we get the principle that “if the omitted facts of a report might substantially alter an impression of the subject which would be congruent with the included facts, then it is unethical.” Congruent is thrown in to try and confuse you, but essentially just means consistent, i.e non-contradictory. This answer is incorrect, it greatly weakens our journalist’s conclusion because she did omit facts that would alter a reader’s impression of the government. I think it is safe to say that if you found out a government was starving people, your impression of it would be altered.

Answer Choice (B) This answer also begins with the general principle “it is ethical to report known facts” which does nothing to help us and should be ignored. Like A the rest of the answer only tells us the conditions for being unethical. This answer weakens what we wish to support, because it is true that the journalist didn’t report facts she knew which might exonerate other factors (like the rebels and nature) by assigning responsibility for the refugees starvation to the government.

Answer Choice (C) If this were a weakening question, this would be a great answer because it necessarily excludes the journalist’s action from being ethical, seeing as she filed a report which a censor had deleted unfavorable material from. But this is a strengthening question, and therefore C is incorrect.

Correct Answer Choice (D) This answer includes an if statement within an unless statement. If we translate it, we can get “If there is a censorship warning, then it is not true that if you report censored material, it is unethical.” This supports the journalist’s conclusion by supporting her reasoning that a censorship notice negates any ethical issues with publishing censored material.

Answer Choice (E) You know what to omit here. This answer is mostly a repeat of D, but with the addition of an AND statement. To be ethical a censored report’s reported facts (i.e the ones that weren’t censored) must now also not give a misleading impression by themselves. This answer is essentially a combination of A and D, and is incorrect for the same reason that A is. The reported facts by themselves, independent from the censored facts and the censorship notice, do give a misleading impression of the government. If you just read the facts that were included, you wouldn’t get the impression that the government was starving people.


Comment on this

This page shows a recording of a live class. We're working hard to create our standard, concise explanation videos for the questions in this PrepTest. Thank you for your patience!

This is a weakening question, we know this because the question stem asks: Which one of the following, if true, weakens the researchers’ argument?

This stimulus has the common LSAT trope of “some people used to believe X, now they believe Y, because of Z.” Our job is to weaken Z. Before scientists assumed that all Dinosaurs were cold-blooded like modern reptiles, but now they believe some may have been warm-blooded because there are fossils in the northern arctic and it is too cold there for cold-blooded animals. The conclusion is a hypothesis meant to explain how dinosaur fossils could be found in the arctic if cold-blooded animals can’t survive the winter there. This argument has some large assumptions it makes to reach the warm-blooded hypothesis; for example, that the dinosaurs remained in the arctic all year long, and that the climate was the same then as it is now. A good answer is going to pick up on one of these assumptions and undermine it, directly weakening the relationship between the support and the conclusion. Let’s take a look at the answer choices:

Answer Choice (A) This supports the argument. Since the stimulus tells us that all of today's reptiles are cold blooded, all this answer adds is that most are confined to particularly warm climates, further strengthening the relationship between cold-blooded animals and warm climates.

Answer Choice (B) This answer brings in size, hoping to get you thinking about how size might relate to the ability to survive in cold environments. The truth is that we just aren’t given any information about how size might relate to the scientist's conclusion, and therefore are not entitled to make any inferences that might weaken the argument.

Answer Choice (C) If the stimulus claimed that Dinosaurs couldn’t have lived in the arctic because there was nothing for them to eat, then the existence of cold-resistant plants might be relevant; but it specifically states that the issue is that cold-blooded animals would freeze to death in an arctic winter. This answer is particularly bad because the plants having been cold resistant further suggests that the arctic was extremely cold in the past as well, which is an assumption that supports the scientists argument.

Correct Answer Choice (D) This answer picks up on one of the assumptions we identified in the stimulus; that the dinosaurs remained in the arctic in all seasons. The researchers’ support specifically states that it is the arctic winters that are the problem. A fossil does not tell us in what season the animal died and whether it lived all year in the area in which it died. This answer undermines this assumption that the dinosaurs lived in the arctic during the winter, from which the researcher’s draw their warm-blooded hypothesis. If the dinosaurs would need to continually migrate, we have a reason to believe they may not have remained in the arctic during the winter and hence may have been cold-blooded like all other reptiles.

Answer Choice (E) This would be a good answer if it were a strengthening question. Like D it picks up on an assumption the argument makes, in this case that the arctic climate was similar during the period of the Dinosaurs. By explicitly stating that it was in fact similar, it directly strengthens the warm-blooded hypothesis.


1 comment